Given an infinite, 2-d, square grid of 1 Ohm resistors, what is the resistance between two adjacent nodes? (Something like a very large window screen, where the wires have finite resistance, but no contact resistance.)
I was given this problem in a math competition (I think it was the Putnam Competition) many years ago, so it must have an answer that can be derived in a finite amount of time with a finite amount of intelligence, but I have never been able to figure it out.
My best idea has been to consider the case of a uniform conductive sheet. I couldn't get it to work, but maybe it will give somebody else a better idea.
At large scales, the sheet will behave the same as the grid if the surface resistivity is equal to the resistance, $R$, of the individual resistors (here 1 Ohm). A current, $I$, fed in at one point will disperse radially with current density equal to $I/(2\pi r)$. Therefore the electric field will be IR/(2pir), and the potential difference between two radii will be equal to $\phi_2 - \phi_1 = \frac{IR}{2\pi}\cdot\ln(r_2/r_1)$.
In this approximation, if the nodes are at $(x=i+0.5d, y=j\cdot d)$, then the Cartesian distance from $(i=\pm0.5, j=0)$ to $(i, j)$ is $(i\pm 0.5)^2 + j^2$. If current is fed into node $a$ and the same current is extracted from node $b$, the solution will be a superposition of the two individual solutions.
$\phi(i, j) = \frac{IR}{2\pi}\cdot\ln(r_{2a}/r_{1a}) - \frac{IR}{2\pi}\cdot\ln(r_{2b}/r_{1b}) = \frac{IR}{2\pi}\cdot\ln\left(\frac{r_{2a}\cdot r_{1b}}{r_{1a}\cdot r_{2b}}\right)$
Let the reference point 2 be on the line of symmetry, where the potential must be zero, so that $r_{2a} = r_{2b}$.
$\phi(i, j) = \frac{IR}{2\pi}\cdot\ln(r_{1b}/r_{1a}) = \frac{IR}{2\pi}\cdot\ln\left(\frac{(i+0.5)^2 + j^2}{(i-0.5)^2 + j^2}\right)$
If this is really a solution, then current continuity at each node requires that the potential at the node be the mean of the potential at the 4 adjacent nodes.
$\ln\left(\frac{(i+0.5)^2 + j^2}{(i-0.5)^2 + j^2}\right) = \ln\left(\frac{(i-0.5)^2 + (j-1)^2}{(i-1.5)^2 + (j-1)^2}\right) + \ln\left(\frac{(i-0.5)^2 + (j+1)^2}{(i-1.5)^2 + (j+1)^2}\right) + \ln\left(\frac{(i+1.5)^2 + (j-1)^2}{(i+0.5)^2 + (j-1)^2}\right) + \ln\left(\frac{(i+1.5)^2 + (j+1)^2}{(i+0.5)^2 + (j+1)^2}\right)$
If you haven't noticed yet, this is not going to get me to a solution. Who has a better idea?
This question should be well-known and easy to find on the internet.
related pages are for example;
http://www.mathpages.com/home/kmath668/kmath668.htm
https://physics.stackexchange.com/questions/2072/on-this-infinite-grid-of-resistors-whats-the-equivalent-resistance